Search results

  • ...use it is both the median and the mode of the set. Thus <math>90</math> is correct.
    2 KB (268 words) - 18:19, 27 September 2023
  • ...idate. Multiplying <math>63\cdot64</math> confirms that our assumption is correct.
    2 KB (315 words) - 15:34, 18 June 2022
  • ...l area gives us <math>40-33\frac{1}{2}=\boxed{6\frac{1}{2}}</math>, so the correct answer is <math>\boxed{\textbf{(D)}}</math>.
    8 KB (1,016 words) - 00:17, 31 December 2023
  • ...ath>1</math> and <math>2016</math>, inclusive. Which of the following is a correct statement about the probability <math>p</math> that the product of the thre
    2 KB (297 words) - 14:54, 25 June 2023
  • ...ectly answer the question receives one point. The person who gets the most correct out of three questions (not necessarily two out of three) is the winner. ...ritten competitors. Starting then, the first person to get three questions correct wins (as opposed to the best-out-of-three rule).
    10 KB (1,497 words) - 11:42, 10 March 2024
  • ...econd minute 4 points, or third minute 3 points. The first team to get the correct answer receives a bonus of 2 points. ...rs are worth 2 points each. Bonus points are given if all four answers are correct and the answer sheet is submitted at the 3 minute mark (First Command – 5
    8 KB (1,182 words) - 14:26, 3 April 2024
  • ...ation competition, a mental math competition (where only exact answers are correct), and a physical estimation competition. Grades 4-9 compete in this format.
    2 KB (216 words) - 16:59, 17 December 2016
  • A correct solution would be to move everything to the left side of the inequality, an
    12 KB (1,798 words) - 16:20, 14 March 2023
  • ...blems are graded on a scale from 0 to 5. The full 5 points are awarded for correct, concise solutions, and points are taken off from there for mistakes or oth
    4 KB (613 words) - 13:08, 18 July 2023
  • The AMC 10 is scored in a way that penalizes guesses. Correct answers are worth 6 points, incorrect questions are worth 0 points, and una
    4 KB (574 words) - 15:28, 22 February 2024
  • The AMC 12 is scored in a way that penalizes guessing. Correct answers are worth 6 points, incorrect answers are worth 0 points, and unans
    4 KB (520 words) - 12:11, 13 March 2024
  • ...ve, making guessing almost futile. Wrong answers receive no credit, while correct answers receive one point of credit, making the maximum score 15. Problems ...inally, additions to and improvements on the solutions in the AoPSWiki are always welcome.
    8 KB (1,057 words) - 12:02, 25 February 2024
  • *4 points for each correct question *5 points for each correct question
    4 KB (632 words) - 17:09, 11 October 2020
  • The test is scored as 5 points for every correct response, 1 point for a blank response, and 0 points for an incorrect respo
    972 bytes (141 words) - 11:12, 30 September 2018
  • ...pping the problem can help you check and solve other problems, and you can always return to the problem that you skipped. This concept helps double where yo ...good guess, saving you time and giving you a chance of getting the answer correct. This tactic helps the most in competitions with multiple choice answers. I
    3 KB (538 words) - 13:13, 16 January 2021
  • ...e triangle. It is easy to verify that this placement of the orthocenter is correct and that the orthic triangle will remain the same as before the swapping, a
    8 KB (1,408 words) - 11:54, 8 December 2021
  • ...rcount several numbers, such as 12, which is divisible by both 2 and 3. To correct for this overcounting, we must subtract out the numbers that are divisible Another basic example is combinations. In these, we correct for overcounting with division, by dividing out what we overcount (as oppos
    4 KB (635 words) - 12:19, 2 January 2022
  • * 4 points for each correct answer ...consists of 5 short-answer questions. Your score is 5 times the number of correct answers, for a maximum score of 25.
    4 KB (644 words) - 12:56, 29 March 2017
  • Each correct answer will earn 5 points, a question left blank will earn 1 point, and an
    2 KB (366 words) - 22:59, 17 November 2007
  • ...> [[factor]]s, two of them are small. If we want to make sure that this is correct, we could test with a smaller number, like <math>30</math>. It becomes much
    2 KB (276 words) - 05:25, 9 December 2023
  • ...le (inclusive and integers only). Full credit is only given for complete, correct solutions. Each solution is intended to be in the form of a [[proof writin
    3 KB (490 words) - 03:32, 23 July 2023
  • For every correct answer: 5 points
    3 KB (388 words) - 23:07, 5 February 2024
  • ...inally, additions to and improvements on the solutions in the AoPSWiki are always welcome.
    3 KB (391 words) - 16:00, 21 February 2024
  • On the AMC 12, each correct answer is worth <math>6</math> points, each incorrect answer is worth <math
    13 KB (1,953 words) - 00:31, 26 January 2023
  • ...ssumes he is the first to arrive. If each takes what he believes to be the correct share of candy, what fraction of the candy goes unclaimed?
    13 KB (1,955 words) - 21:06, 19 August 2023
  • ..., and <math>R</math> the area of the red square. Which of the following is correct?
    12 KB (1,792 words) - 13:06, 19 February 2020
  • Cassandra sets her watch to the correct time at noon. At the actual time of 1:00 PM, she notices that her watch rea
    13 KB (1,987 words) - 18:53, 10 December 2022
  • Inductive Step: Suppose the formula is correct for <math>z_k</math>, then ...z|</math>. Therefore, the magnitude of <math>\frac{iz_n}{|z_n|}</math> is always <math>1</math>, meaning that all of the numbers in the sequence <math>z_k</
    4 KB (660 words) - 17:40, 24 January 2021
  • ...4}</math> But, this over-counts since it counts numbers like 0213. We can correct for this over-counting. Lock the first digit as 0 and permute 3 other chose
    3 KB (562 words) - 18:12, 4 March 2022
  • ...clear from the problem setup that <math>0<\theta<\frac\pi2</math>, so the correct value is <math>\tan(\theta)=\frac53</math>. Next, extend rays <math>\overri
    9 KB (1,501 words) - 05:34, 30 October 2023
  • ...the formula <math>s=30+4c-w</math>, where <math>c</math> is the number of correct answers and <math>w</math> is the number of wrong answers. Students are not
    6 KB (933 words) - 01:15, 19 June 2022
  • ...ailable ten-button lock may be opened by depressing -- in any order -- the correct five buttons. The sample shown below has <math>\{1, 2, 3, 6, 9\}</math> as
    6 KB (902 words) - 08:57, 19 June 2021
  • ...bsent-minded professor failed to notice that his calculator was not in the correct angular mode. He was lucky to get the right answer. The two least positive
    7 KB (1,177 words) - 15:42, 11 August 2023
  • ...the formula <math>s=30+4c-w</math>, where <math>c</math> is the number of correct answers and <math>w</math> is the number of wrong answers. (Students are no Let Mary's score, number correct, and number wrong be <math>s,c,w</math> respectively. Then
    7 KB (1,163 words) - 23:53, 28 March 2022
  • Since this is an AIME problem, there is exactly one correct answer, and thus, exactly one possible value of <math>f(14,52)</math>.
    4 KB (538 words) - 13:24, 12 October 2021
  • ...available ten-button lock may be opened by pressing -- in any order -- the correct five buttons. The sample shown below has <math>\{1,2,3,6,9\}</math> as its
    1 KB (181 words) - 18:23, 26 August 2019
  • ...As no other option choice fits, <math>\boxed{\textbf{(A)}-x}</math> is the correct solution.
    1 KB (179 words) - 10:33, 19 August 2022
  • THIS SOLUTION IS INCORRECT, PLEASE CORRECT IT IF YOU HAVE TIME!
    3 KB (447 words) - 17:02, 24 November 2023
  • ...r of <math>11</math> is <math>(5,11)</math>, and checking shows that it is correct.
    4 KB (628 words) - 22:05, 7 June 2021
  • ...way to do the problem is by the process of elimination. The only possible correct choices are the highest powers of each prime, <math>2^3=8</math>, <math>3^2
    5 KB (878 words) - 14:39, 3 December 2023
  • ...th> to <math>96</math> alternate in fake-real-fake-real, where we have the correct order of cards once the first <math>96</math> have moved and we can start p ...the spacing of the cards moved, <math>a</math> is an integer such that the correct first card is moved, and <math>k</math> is an integer greater than or equal
    15 KB (2,673 words) - 19:16, 6 January 2024
  • ...ing <math>2</math> and <math>7</math> for <math>w_1</math> does not give a correct product. Thus, <math>\frac{27}{50}</math> must be a reduced form of the ac
    7 KB (1,011 words) - 20:09, 4 January 2024
  • ...ination counts only one of the permutations; we can say that it counts the correct (ascending order) permutation. ...sformations of the problem, a recursion formula can be a robust way to the correct answer.
    11 KB (1,729 words) - 20:50, 28 November 2023
  • ...th> and <math>C</math>. Oh wait they are symmetric. So then if this is the correct answer, why am I wrong, or what happened to that factor of <math>3</math>?"
    15 KB (2,406 words) - 23:56, 23 November 2023
  • ...bsent-minded professor failed to notice that his calculator was not in the correct angular mode. He was lucky to get the right answer. The two least positive
    2 KB (336 words) - 19:30, 24 June 2020
  • ...math>, so no real solution exists for <math>x</math>. Thus our solution is correct.
    6 KB (1,060 words) - 17:36, 26 April 2024
  • ...math>11</math> while <math>(a-b)</math> is a factor of nine (1 or 9). The correct guesses are <math>a = 6, b = 5</math> causing <math>x = 65, y = 56,</math>
    5 KB (845 words) - 19:23, 17 September 2023
  • ...h>, we see <math>6\cdot8=30+18</math>. Therefore, we can see our answer is correct.
    1 KB (155 words) - 17:30, 16 December 2021
  • Thus, the correct answer is <math>\boxed{\textbf{(C) }3}.</math> Note to readers: make sure to always read the problem VERY carefully before attempting; it could mean the differ
    3 KB (450 words) - 02:00, 13 January 2024
  • ...= 72</math>. 28 is not divisible by 3, so we know that this number is not correct. Moving on to 7, <math>13 \cdot 7 = 91</math>. We know that 9 is a multiple
    3 KB (429 words) - 18:14, 26 September 2020

View (previous 50 | next 50) (20 | 50 | 100 | 250 | 500)